Finding The Right Row

Logic Level 3

1 2 2 3 4 3 4 6 6 4 5 8 9 8 5 6 10 12 12 10 6 \begin{array} {ccccccccc} 1 \\ 2 \quad 2 \\ 3 \quad 4 \quad 3 \\ 4 \quad 6 \quad 6 \quad 4 \\ 5 \quad 8 \quad 9 \quad 8 \quad 5 \\ 6 \hspace{0.32cm} 10 \hspace{0.32cm} 12 \hspace{0.32cm} 12 \hspace{0.32cm} 10 \hspace{0.32cm} 6 \\ \vdots \end{array}

In the triangle above, the outer diagonal lines are 1 , 2 , 3 , 4 , , 1,2,3,4,\ldots, which begins at 1 and each number after the first is one larger than the previous number.

The next diagonal lines are 2 , 4 , 6 , 8 , , 2,4,6,8,\ldots, which begins at 2 and each number after the first is two larger than the previous number.

SImiarly, the n th n^\text{th} diagonal begins at n n and each number after the first is n n larger than the previous number.

In which horizontal row does the number 2016 first appear?


The answer is 89.

This section requires Javascript.
You are seeing this because something didn't load right. We suggest you, (a) try refreshing the page, (b) enabling javascript if it is disabled on your browser and, finally, (c) loading the non-javascript version of this page . We're sorry about the hassle.

2 solutions

The triangle shown is in fact an infinite multiplication table of two positive integers a a and b b . The mulytiplands a a and b b are represented by the two sets of diagonals in opposite directions. Let a a be represented by the diagonals from the left and b b , from right. Then the first left diagonal represents a = 1 a=1 ; the second left diagonal, a = 2 a=2 ; the third left diagonal, a = 3 a=3 and so on. Similarly for b b but with the right diagonal. The intersection of the a a -diagonal and b b -diagonal is the product n = a b n=ab . For example, 1 × 6 = 6 1 \times 6 = 6 is the intersection of the first a a -diagonal and the sixth b b -diagonal. While 2 × 3 = 6 2 \times 3 = 6 is the intersection of the second a a -diagonal and the third b b -diagonal.

We note that when a a or b b increases by 1, the row number r r increases by 1. Now, let us assign the intersection be ( a , b ) (a,b) . To get to the intersection n = a b n=ab , we go from ( 1 , 1 ) (1,1) \rightarrow ( 2 , 1 ) (2,1) \rightarrow ( 3 , 1 ) (3,1) \rightarrow . . . ... \rightarrow ( a , 1 ) (a,1) and then from ( a , 1 ) (a,1) \rightarrow ( a , 2 ) (a,2) \rightarrow ( a , 3 ) (a,3) \rightarrow . . . ... \rightarrow ( a , b ) (a,b) . At ( 1 , 1 ) (1,1) , r = 1 r=1 ; at ( a , 1 ) (a,1) , r = a r=a ; at ( a , 2 ) (a,2) , r = a + 1 r=a+1 . . . ... at ( a , b ) (a,b) , r = a + b 1 r=a+b-1 .

Without loss of generality, let us assume b > a b>a and let δ = b a \delta=b-a be the difference between the two divisors a a and b b of n n . Then, we have:

a b = n a ( a + δ ) = n a 2 + δ a n = 0 a = δ + δ 2 + 4 n 2 As a > 0 , the negative root is rejected. \begin{aligned} ab & = n \\ a(a+\delta) & = n \\ a^2 + \delta a -n & = 0 \\ \implies a & = \frac {-\delta+\sqrt{\delta^2+4n}}2 & \small {\color{#3D99F6}\text{As }a > 0 \text{, the negative root is rejected.}} \end{aligned}

Now let us find the minimum r r . Since r = a + b 1 r=a+b-1 , let us find the minimum a + b a+b .

a + b = 2 a + δ = δ + δ 2 + 4 n + δ = δ 2 + 4 n \begin{aligned} a+b & = 2a + \delta \\ & = -\delta+\sqrt{\delta^2+4n} + \delta \\ & = \sqrt{\delta^2+4n} \end{aligned}

So we can see that a + b a+b increases with δ \delta , the difference between a a and b b . And for all positive real a a and b b , a + b a+b is minimum when δ = 0 \delta = 0 which is proven by AM-GM inequality : a + b 2 a b = 2 n a+b \ge 2\sqrt{ab} = 2\sqrt n . For integral a a and b b , minimum a + b a+b occurs when δ = b a \delta = b-a is minimum. For n = 2016 n=2016 , a = 42 a=42 and b = 48 b=48 and r = 42 + 48 1 = 89 r=42+48-1 = \boxed{89} .

Nice solution. Thank you for the solution. I solved it in one page and was worried how am I gonna write all this using Latex and here you solved it in one paragraph nicely and logically. Thanks again :)

Hana Wehbi - 4 years, 7 months ago

Log in to reply

You are welcome. I did the triangle in LaTex too.

Chew-Seong Cheong - 4 years, 7 months ago

Log in to reply

Thank you, it looks much better now :)

Hana Wehbi - 4 years, 7 months ago

The solution contains the correct steps, and I have some suggestions for clarity. Presenting a clear solution is quite different from simply solving the problem. You have to clearly explain to others, and not just yourself.

  1. First explain what each row is. In particular, row n n entry r r is r ( n + 1 r ) r (n+1-r) .
  2. Next, explain that "Hence, we can determine which rows the positive integers x = p q x = pq will appear in, as "row p+q+1, entry p".
  3. Lastly, explain the "minimum over integers" properly. We cannot use AM-GM. A better explanation (but still not perfect) would be to say "We want to find the minimum of p + 2016 p + 1 p + \frac{2016}{p} + 1 . By considering the graph, we want p p to be as close to 2016 \sqrt{2016} as possible

Calvin Lin Staff - 4 years, 7 months ago

Log in to reply

I thought the comment was for me. Sir @Chew-Seong Cheong , I found your solution nice and logical. Sorry for the confusion.

Hana Wehbi - 4 years, 7 months ago

Log in to reply

It is okay. Calvin meant well.

Chew-Seong Cheong - 4 years, 7 months ago

I agree, thank you.

Hana Wehbi - 4 years, 7 months ago

Sorry, I thought it was quite obvious and maybe it was late here then. Anyway, I have redone the solution.

Chew-Seong Cheong - 4 years, 7 months ago

Nice learned something :P

anukool srivastava - 4 years, 1 month ago
Kai Ott
Nov 10, 2016

A perhaps shorter but more arbitrary and impractical solution. We know that 2016 appears in every line n = 2016 x + x 1 n = \frac{2016}{x} + x -1 where is the line from which you start going inwards the triangle. With calculus we find the minimun value n 88.8 n \approx 88.8 , but of course n needs to be a natural numbers so we get a theoretical minimum of 89. Now we see if that can be achieved. Solve for x in the above equation which yields x = 42 x=42 . Thus the theoretical minumun can be achieved hence n = 89 n=89 .

A perhaps even more "Crude" way. Note that provided n|2016, following the nth diagonal, 2016 appears on the [(2016/n)+n-1]th row. So for example, since 1|2016, 2016 appears on the 2016th row. Since 2|2016, 2016 appears not the 1009th row, and so on. Now, list all the divisors of 2016 and check which row they imply 2016 occurs in. The minimum of these rows is your target. (After some hard work, you find that 42|2016 implying 2016 occurs at the 89th row is the minimum).

Ceesay Muhammed - 4 years, 7 months ago

The nth row will have the following n numbers;.
n,(n-1)(2),(n-2)(3),..(n-k)(k+1)...{n-(n-1)(n) The idea is to find (n-k)(k+1)=2016,.
such that (n-k)+(k+1)=(n+1) is minimum. The sum would be minimum when (n-k) and (k+1) are as close to each other as possible, since 2016 is not perfect square. 2016=48×42 gives the two closest factors of 42 and 48, which can be written as, (89-47)(48) or (89-41)(42), which means n=89th row.

Auro Light - 3 years, 9 months ago

0 pending reports

×

Problem Loading...

Note Loading...

Set Loading...